LSAT and Law School Admissions Forum

Get expert LSAT preparation and law school admissions advice from PowerScore Test Preparation.

 Administrator
PowerScore Staff
  • PowerScore Staff
  • Posts: 8917
  • Joined: Feb 02, 2011
|
#80560
Complete Question Explanation

Justify-FIB, FL. The correct answer choice is (D).

Answer choice (A):

Answer choice (B):

Answer choice (C):

Answer choice (D): This is the correct answer choice.

Answer choice (E):

This explanation is still in progress. Please post any questions below!
 Kevin63
  • Posts: 2
  • Joined: Nov 06, 2020
|
#80800
Why is the answer D and not C?

I had the following logic:

Premise A: 17 -(most)-> computer programmers
Missing Premise: Gov. Employees in Hansen Building -(most)-> 17
_______________
Conclusion: Gov. Employees in Hansen Building -(some)-> computer programmers

Wouldn't this connect everything?
 lsac_noob_89
  • Posts: 4
  • Joined: Nov 06, 2020
|
#80812
Hi! you cannot flow two most statements from left to right like you can combine two "all" statements.

For ex)

A most ---> C
A most ---> B
results in
Some C ---> B (as well as Some B ---> C)

Therefore we were looking for a
17 most ---> Hanson which is answer choice D

17 most ---> computer programmers
17 most ---> Hanson
Some computer programmers ---> Hanson which is reversible to to Some Hanson ---> computer programmers
 lsac_noob_89
  • Posts: 4
  • Joined: Nov 06, 2020
|
#80813
To clarify combining left to right statements -

Most X ---> Y
Most Y ---> Z

^ there is no valid conclusion you can draw from this.

For an example)
Most Apples ---> Red
Most Red ---> Expensive

Let's say that we have 10 apples, and "most" implies that 6 of them are red.
However, it could be the case that we have 1,000 red items, and more than 50% are expensive.
The 6 red apples could have zero overlap with the 501 (more than 50%) expensive red items.
 lsac_noob_89
  • Posts: 4
  • Joined: Nov 06, 2020
|
#80814
Also remember "Most" in LSAT world is defined as greater than 50%. It's not like common usage which implies a plurality.
 lsac_noob_89
  • Posts: 4
  • Joined: Nov 06, 2020
|
#80815
lsac_noob_89 wrote:Also remember "Most" in LSAT world is defined as greater than 50%. It's not like common usage which implies a plurality.
*implies a majority
 Kevin63
  • Posts: 2
  • Joined: Nov 06, 2020
|
#80820
Thank you very much for the explanation!
 Paul Marsh
PowerScore Staff
  • PowerScore Staff
  • Posts: 290
  • Joined: Oct 15, 2019
|
#80852
Very nicely explained lsac_noob! Just to add onto that - on the LSAT, the only time that two "most" statements can yield a combination inference is when they begin with the same group. For example: "Most Texans own a house. Most Texans own a horse." We can draw the valid inference from those two statements that at least some Texans own both a house and a horse. The reason we can make that inference is simple math - if at least 51/100 own a house and at least 51/100 own a horse, there must be at least one with both. But if I don't have two "most" statements that begin with the same group, I won't be able to make any inferences like that.

Kevin, if you have any further questions about this one feel free to follow up below!
 kenlars5
  • Posts: 21
  • Joined: Oct 27, 2020
|
#83096
Paul Marsh wrote: Mon Nov 09, 2020 5:23 am Very nicely explained lsac_noob! Just to add onto that - on the LSAT, the only time that two "most" statements can yield a combination inference is when they begin with the same group. For example: "Most Texans own a house. Most Texans own a horse." We can draw the valid inference from those two statements that at least some Texans own both a house and a horse. The reason we can make that inference is simple math - if at least 51/100 own a house and at least 51/100 own a horse, there must be at least one with both. But if I don't have two "most" statements that begin with the same group, I won't be able to make any inferences like that.

Kevin, if you have any further questions about this one feel free to follow up below!
So then its correct for me to approach this question like this??
P: BUN17 :most: CP

C: HB :some: CP

Prephrase:
BUN17 —> HB
Because you could turn the most statement into a some CP :some: BUN17 —> HB = CP :some: HB

OR
BUN17 :most: HB because you can connect 2 mosts that both lead away from a common middle, HB <—(most) BUN17 :most: CP = HB :some: CP

Just want to make sure because I was a little confused by the explanation.
 theamazingrace
  • Posts: 59
  • Joined: Oct 17, 2020
|
#83166
This is how I got D:
BUN17 :most: CP :some: HSB

I see people talking about two mosts to come up with D, what am I missing?

Thanks!

Get the most out of your LSAT Prep Plus subscription.

Analyze and track your performance with our Testing and Analytics Package.